Hecke Bound for Cusp - Modular Forms

  • Thread starter Thread starter binbagsss
  • Start date Start date
  • Tags Tags
    Bound Forms
Click For Summary
The discussion focuses on a question regarding the Hecke bound for cusp forms in modular forms, specifically addressing a misunderstanding related to Proposition 2.12. The participant acknowledges that their reasoning about the behavior of the function as v approaches infinity may be flawed, particularly in relation to the dependency on the cusp condition. They express confusion about how the exponential term dominates the polynomial term in their calculations. The participant seeks clarification on where their reasoning went wrong, emphasizing the need for a correct understanding of the behavior of q in the upper half-plane. The conversation highlights the complexities involved in analyzing cusp forms and their properties in modular forms.
binbagsss
Messages
1,291
Reaction score
12

Homework Statement



i have a questions on the piece of lecture notes attached:

whatthehecke.png

2. Homework Equations

The Attempt at a Solution


[/B]

I agree 2) of proposition 2.12 holds once we have 1). I thought I understood the general idea of 1), however, my reasoning would hold for ##M_k## it does not depend on ##f(t)## being a cusp and so it must be wrong. This was what I thought was happening:

##q=e^{2\pi i n (u+iv)} ~ e^{-v} ## for large v, and exponential dominates over ##v ^ {x}## ( v>0 as on upper plane )

This would ofc still hold if I included some constant term, I would still get the same quantity is bounded.

can someone please tel me where I have gone wrong with the above reasoning?

 

Attachments

  • whatthehecke.png
    whatthehecke.png
    30.7 KB · Views: 814
Physics news on Phys.org
binbagsss said:
##q=e^{2\pi i n (u+iv)} \approx e^{-v} ## for large v, and exponential dominates over ##v ^ {x}## ( v>0 as on upper plane )



edited apologies latex error
 
Question: A clock's minute hand has length 4 and its hour hand has length 3. What is the distance between the tips at the moment when it is increasing most rapidly?(Putnam Exam Question) Answer: Making assumption that both the hands moves at constant angular velocities, the answer is ## \sqrt{7} .## But don't you think this assumption is somewhat doubtful and wrong?

Similar threads

Replies
9
Views
2K
  • · Replies 3 ·
Replies
3
Views
2K
  • · Replies 5 ·
Replies
5
Views
2K
Replies
20
Views
2K
Replies
5
Views
3K
  • · Replies 14 ·
Replies
14
Views
3K
  • · Replies 2 ·
Replies
2
Views
1K
Replies
7
Views
2K
  • · Replies 3 ·
Replies
3
Views
1K
  • · Replies 5 ·
Replies
5
Views
4K